Phương sai tối đa của một mẫu là gì?


13

B

Var(maxiXi)B,
X={X1,,XM}Mμ1,,μMσ12,,σM2

Tôi có thể suy luận rằng

Var(maxiXi)iσi2,
nhưng ràng buộc này có vẻ rất lỏng lẻo. Một bài kiểm tra số dường như chỉ ra rằng B=maxiσi2 có thể là một khả năng, nhưng tôi chưa thể chứng minh điều này. Bất kỳ trợ giúp được đánh giá cao.

3
(Bạn có muốn giả sử Xi là độc lập không?) Giả thuyết này hợp lý nhưng có vẻ là sai. Chẳng hạn, thực hiện một số thử nghiệm trong đó Xi là iid với CDF 1x1s , 1x , s>3 . Phương sai tối đa của chúng, liên quan đến phương sai chung của chúng, tăng mà không bị ràng buộc khi M phát triển.
whuber

@whuber Cảm ơn, điều đó giải thích tại sao tôi không thể chứng minh phỏng đoán đó :) Tôi thực sự quan tâm đến trường hợp Xi độc lập. Chỉ cần làm rõ, tôi chủ yếu quan tâm đến giới hạn chung chỉ sử dụng hai khoảnh khắc đầu tiên. Tôi không chắc liệu giới hạn chung sắc nét hơn có tồn tại hơn phương sai chung hay không.
Peter

1
Tôi nên chỉ ra rằng tổng của bạn bị ràng buộc (giả sử nó là chính xác - thật tuyệt khi thấy một bản phác thảo bằng chứng) là chặt chẽ. Chẳng hạn, hãy để được hỗ trợ trong khoảng với phương sai không vượt quá và để được hỗ trợ trên . Sau đó , với phương sai , nhưng bất đẳng thức có thể được thắt chặt bao nhiêu tùy thích bằng cách thu nhỏ . [ - , một ] ε 2 X 1 [ một , ] max i X i = X 1 σ 2 1σ 2 1 + ( M - 1 ) ε 2 ε 2X2,,XM[,a]ε2X1[a,]maxiXi=X1σ12σ12+(M1)ε2ε2
whuber

1
Đối với dữ liệu iid, lý thuyết giá trị cực trị cung cấp các lớp phân phối mà mẫu tối đa hội tụ, với các điều kiện nhất định trên các đuôi của phân phối ban đầu tạo ra các lớp phân phối tiệm cận khác nhau. Vì vậy, tôi nghi ngờ rằng bạn sẽ có thể rút ra một ràng buộc tốt chỉ dựa trên hai khoảnh khắc, mặc dù tôi chỉ quen thuộc với lý thuyết.
StasK

Câu trả lời:


9

Đối với bất kỳ biến ngẫu nhiên , ràng buộc chung tốt nhất là như đã nêu trong câu hỏi ban đầu. Dưới đây là một bản phác thảo bằng chứng: Nếu X, Y là IID thì . Cho một vectơ gồm các biến phụ thuộc có thể , hãy để là một vectơ độc lập có cùng phân phối chung. Với bất kỳ , chúng ta có liên kết ràng buộc rằng và tích hợp này từ đến mang lại sự bất bình đẳng được yêu cầu.X i V một r ( max X i ) Σ i V một r ( X i ) E [ ( X - Y ) 2 ] = 2 V một r ( X ) ( X 1 , ... , X n ) ( Y 1 , ... , Y n ) r > 0 P [nXiVar(maxXi)iVar(Xi)E[(XY)2]=2Var(X)(X1,,Xn)(Y1,,Yn)r>0d r 0 P[|maxiXimaxiYi|2>r]iP[|XiYi|2>r]dr0

Nếu là chỉ số IID của các sự kiện có xác suất , thì là chỉ báo về sự kiện có xác suất . Khắc phục và để có xu hướng về 0, chúng tôi nhận được và . ϵ max X i n ϵ + O ( n 2 ϵ 2 ) n ϵ V a r ( X i ) = ϵ - ϵ 2 V a r ( max i X i ) = n ϵ + O ( n 2 ϵ 2 )XiϵmaxXinϵ+O(n2ϵ2)nϵVar(Xi)=ϵϵ2Var(maxiXi)=nϵ+O(n2ϵ2)


3

Một câu hỏi trên MathOverflow có liên quan đến câu hỏi này.

Đối với các biến ngẫu nhiên IID, cao thứ được gọi là thống kê đơn hàngk .

Ngay cả đối với các biến ngẫu nhiên của IID Bernoulli, phương sai của bất kỳ thống kê đơn hàng nào ngoài trung bình có thể lớn hơn phương sai của dân số. Ví dụ: nếu là với xác suất và với xác suất và , thì tối đa là với xác suất , do đó phương sai của dân số là trong khi phương sai của tối đa là khoảng .Xi11/1009/10M=10111/e0.090.23

Đây là hai bài báo về phương sai của thống kê đơn hàng:

Yang, H. (1982) "Về phương sai của trung vị và một số thống kê đơn hàng khác." Bò đực. Inst. Môn Toán. Học viện Sinica, 10 (2) trang 197-204

Papadatos, N. (1995) "Phương sai tối đa của thống kê đơn hàng." Ann. Inst. Thống kê. Toán., 47 (1) trang 185-193

Tôi tin rằng giới hạn trên của phương sai tối đa trong bài báo thứ hai là . Họ chỉ ra rằng sự bình đẳng không thể xảy ra, nhưng bất kỳ giá trị thấp hơn nào cũng có thể xảy ra đối với các biến ngẫu nhiên IID Bernoulli.Mσ2

Khi sử dụng trang web của chúng tôi, bạn xác nhận rằng bạn đã đọc và hiểu Chính sách cookieChính sách bảo mật của chúng tôi.
Licensed under cc by-sa 3.0 with attribution required.